Difference between revisions of "2018 AMC 10A Problems/Problem 1"

m
Line 2: Line 2:
 
<cmath>\left(\left((2+1)^{-1}+1\right)^{-1}+1\right)^{-1}+1?</cmath><math>\textbf{(A) } \frac58 \qquad \textbf{(B) }\frac{11}7 \qquad \textbf{(C) } \frac85 \qquad \textbf{(D) } \frac{18}{11} \qquad \textbf{(E) } \frac{15}8 </math>
 
<cmath>\left(\left((2+1)^{-1}+1\right)^{-1}+1\right)^{-1}+1?</cmath><math>\textbf{(A) } \frac58 \qquad \textbf{(B) }\frac{11}7 \qquad \textbf{(C) } \frac85 \qquad \textbf{(D) } \frac{18}{11} \qquad \textbf{(E) } \frac{15}8 </math>
  
== Solution
+
== Solution ==
 
Arithmetic gives <math>(\textbf{B})</math>
 
Arithmetic gives <math>(\textbf{B})</math>
  
 
== See Also ==
 
== See Also ==
  
{{AMC10 box|year=2018|ab=A|num-b=9|num-a=11}}
+
{{AMC10 box|year=2018|ab=A|num-a=2}}
 
{{MAA Notice}}
 
{{MAA Notice}}

Revision as of 17:34, 8 February 2018

What is the value of \[\left(\left((2+1)^{-1}+1\right)^{-1}+1\right)^{-1}+1?\]$\textbf{(A) } \frac58 \qquad \textbf{(B) }\frac{11}7 \qquad \textbf{(C) } \frac85 \qquad \textbf{(D) } \frac{18}{11} \qquad \textbf{(E) } \frac{15}8$

Solution

Arithmetic gives $(\textbf{B})$

See Also

2018 AMC 10A (ProblemsAnswer KeyResources)
Preceded by
[[2018 AMC 10A Problems/Problem {{{num-b}}}|Problem {{{num-b}}}]]
Followed by
Problem 2
1 2 3 4 5 6 7 8 9 10 11 12 13 14 15 16 17 18 19 20 21 22 23 24 25
All AMC 10 Problems and Solutions

The problems on this page are copyrighted by the Mathematical Association of America's American Mathematics Competitions. AMC logo.png